what is cosec32°/sec58°=

Answers

Answer 1

Answer:

1

Step-by-step explanation:

Using the cofunction identity

sec(90 - x) = cosecx , then

sec(90 - 58)° = cosec32° , so

[tex]\frac{cosec32}{sec58}[/tex]

= [tex]\frac{cosec32}{cosec32}[/tex]

= 1


Related Questions

AB is a line segment.
The midpoint of the line segment AB has coordinates (3, 5)

Point A has coordinates (9, 2)
Work out the coordinates of point B.

Answers

Answer:

(-3,8)

Step-by-step explanation:

Lets say two points are (x1,y1) and (x2,y2), the midpoint will be:

((x1+x2)/2,(y1+y2)/2)

Plugging the point A in, we get: the midpoint, (3,5) as (((9+x2)/2),(2+y2)/2)

We can assign x2 and y2 as variables.

(9+x2)/2=3

9+x2=6

x2= -3

(2+y2)/2=5

2+y2=10

y2=8

therefore we get the point (-3,8)

There are better ways and this is the "formulaic (ig) way" to do this

The midpoint of a line segment means a point which lies in the mid of the given line segment. The coordinate of point B is (-3,8).

What are the coordinates of the midpoint of the line segment AB?

The midpoint of a line segment means a point which lies in the mid of the given line segment.

Suppose we've two endpoints of a line segment:

A(p,q), and B(m,n)

Then let the midpoint be M(x,y) on that line segment. Then, its coordinates are:

 x = (p+m)/2

and

y = (q+n)/2

The coordinate of the midpoint (x,y)=(3,5), while the coordinate of A(p,q)=(9,2). Thus, the coordinate of point A are,

x=(p+m)/2

3=(9+m)/2

6=9+m

m = -3

y = (q+n)/2

5 = (2+n)/2

10=2+n

n=8

Hence, the coordinate of point B is (-3,8).

Learn more about Midpoint:

https://brainly.com/question/5127660

#SPJ2

A. Express the following in scientific notation 1.0.948901 2.11,000,000 3.0.08907

Answers

Answer:

answer should be c good luck

What is the answer step by step explanation please.

w (3w + 2) + 5w​

Answers

Answer:8w^2+2

w(3w+2)+5w

First distribute the w to the parentheses:

3w^2+w

Add on the 5w to the new equation:

3w^2+2w+5w

Combine like terms:

8w^2+2

Owen is driving to a concert and needs to pay for parking. There is an automatic fee of $12 just to enter the parking lot, and when he leaves the lot, he will have to pay an additional $3 for every hour he had his car in the lot. How much total money would Owen have to pay for parking if he left his car in the lot for 5 hours? How much would Owen have to pay if he left his car in the lot for tt hours?

Answers

Answer:

27$

Step-by-step explanation:

I do not know what you mean by tt hours, but I can create an equation for the problem

Y=3x+12

Essentially, every X hours he spends in the lot you add 3$, but you start off with 12$. Please mark this brainliest or thank it.

Given that T is the centroid of △DEF and FT=12

Answers

The centroid of a triangle divides the median of the triangle into 1 : 2

The measure of FQ is 18, while the measure of TQ is 6

Because point T is the centroid, then we have the following ratio

[tex]\mathbf{TQ : FT =1 : 2}[/tex]

Where FT = 12.

Substitute 12 for FT in the above ratio

[tex]\mathbf{TQ : 12 =1 : 2}[/tex]

Express as fraction

[tex]\mathbf{\frac{TQ }{ 12} =\frac{1 }{ 2}}[/tex]

Multiply both sides by 12

[tex]\mathbf{TQ =\frac{1 }{ 2} \times 12}[/tex]

This gives

[tex]\mathbf{TQ =\frac{1 2}{ 2}}[/tex]

Divide 12 by 2

[tex]\mathbf{TQ =6}[/tex]

The measure of FQ is calculated using:

[tex]\mathbf{FQ = FT + TQ}[/tex]

Substitute 12 for FT, and 6 for TQ

[tex]\mathbf{FQ = 12 + 6}[/tex]

Add 12 and 6

[tex]\mathbf{FQ = 18}[/tex]

Hence, the measure of FQ is 18, while the measure of TQ is 6

Read more about centroids at:

https://brainly.com/question/11891965

13x+y=1
2x+6y=6
HELP PLZ

Answers

Answer:

(0,1)

Step-by-step explanation:

solve the first equation for y. y=1-13x

plug this into the other equation and solve for x.

2x+6(1-13x)=6

2x+6-78x=6

-76x+6=6

subtract 6 from both sides

-76x=0

x=0

plug this into the other original equation.

13(0)+y=1

y=1

(0,1)

A piece of metal with a length of 2. 83 cm was measured using four different devices. Which of the following measurements is the most accurate? 2. 837 cm 2. 829 cm 2. 812 cm 2. 805 cm.

Answers

Answer:

2.829 mm

Step-by-step explanation:

Correct:  2.83 mm

Measured(mm)    Difference (mm)

2.837             0.007

2.829            -0.001  Smallest deviation from actual.

2.812            -0.018

2.805            -0.025

The measurement of the piece of metal that is the most accurate is 2.837 cm.

Given that,

A piece of metal with a length of 2.83 cm was measured using four different devices.

We have to determine,

Which of the following measurements is the most accurate?

According to the question,

A piece of metal with a length of 2.83 cm was measured using four different devices.

Accuracy is the degree to which the measured value is close to the correct value.

The difference between the accepted value and the measured value of the quantity is called the error of measurement.

The measurement for the first device is,

[tex]=2.83 - 2.837\\\\=-0.007[/tex]

The measurement of the first device is -0.007.

The measurement for the second device is,

[tex]=2.83 - 2.829\\\\=-0.001[/tex]

The measurement of the second device is -0.001.

The measurement for the third device is,

[tex]=2.83 - 2.812\\\\=-0.018[/tex]

The measurement of the third device is -0.018.

The measurement for the fourth device is,

[tex]=2.83 - 2.805\\\\=-0.025[/tex]

The measurement of the fourth device is -0.02.

Hence, The measurement that is the most accurate is 2.837 cm.

For more details about Substitution refer to the link given below.

https://brainly.com/question/2785991

What is solution for 7y+9+6y=46

Answers

Answer:

combine like terms

7y + 6y

 13y + 9

Step-by-step explanation:

here is the solution

13y + 9

whats the exact length of BC?

Answers

-..........The  answer is b-

--------------------------------------------------------

(15)²= (9)²+(BC)²

225=81+(BC)²

BC=√144

BC=12

Answer:

B:12

Step-by-step explanation:

The length of BC is 12 because we can use the Pythagorean theorem to solve this. First, we know what the hypotenuse is 15 and the one of the side lengths which is 9. he equation to solve this is h^2-a^2 when h is 15 and a=9. So, we we plug in the info, we get 15^2-9^2 which is 15*15-9*9 which is 225-81 and that is 144. We are not done yet because we need to find the square root of 144 since we squared 15 and 9 in the first place. The √144 is 12 because 12*12 is 144. Now, we have to check! The equation is a^2+b^2=h^2 when a=12, b=9, and h=15! So, it will be 12^2+9^2=15^2 which simplifies to 144+81=225 and that is true!! So thats why it is B. Hope this helps!!!!!!

Plz mark me as brainliest

What is x, the distance between points A and A'? 2. 4 units 4. 8 units 13. 6 units 14. 4 units.

Answers

The distance between points A (3, -5) and A' (2, -3) is 2.4 units.

Given that,

The points A (3, -5) and A' (2, -3).

We have to determine,

The distance between point A and A'.

According  to the question,

The distance between two points is determined by using the distance formula.

[tex]\rm Distance \ formula = \sqrt{(x_2-x_2) ^2+ (y_2-y_2)^2[/tex]

Then,

The distance between points A (3, -5) and A' (2, -3) is,

[tex]\rm Distance = \sqrt{(2-3) ^2+ ((-3)-(-5))^2}\\\\ Distance = \sqrt{(-1) ^2+ (2)^2}\\\\ Distance = \sqrt{1+4}\\\\Distance = \sqrt{5}\\\\Distance = 2.4 \ units[/tex]

Hence, The distance between points A (3, -5) and A' (2, -3) is 2.4 units.

For more details refer to the link given below.

https://brainly.com/question/8069952

Answer:

13.6 units

Step-by-step explanation:

Edge

The distance between two cities is 180 km. The distance was
covered by a car at a rate of 50 km/h, by a bus at 45 km/h, by a
truck at 40 km/h, and a by a bicycle at 25 km/h. What is the constant of
variation?

Answers

Answer:

Step-by-step explanation:

he distance between two cities is 180 km.

Speed = distance/time

v = s/t from this we can calculate for time  

Time = distance /speed

This distance was covered by a car at a rate of 50 km/h

Time taken by the car to cover this distance of 180km

= 180/50 = 3.6 hours  

This distance was covered by a bus at 45 km/h

 

Time taken by the bus to cover this distance of 180km

= 180/45 = 4 hours

This distance was covered by a truck at 40 km/h

Time taken by the truck to cover this distance of 180km

= 180/40 = 4.5 hours

This distance was covered by a bicycle at 25 km/h

Time taken by the truck to cover this distance of 180km

= 180/25 = 7.2 hours

Therefore:

speed and time are directly proportional.

Speed, s is inversely proportional to time, t

xy = k (inverse variation) or y/x = k (direct variation), where k is the constant of variation

Introducing constant of variation, k  

s = k/t  

When s = 25, t = 7.2

25 = k/7.2

k = 7.2× 25 = 180

So

s = 180/k

If 7 out of 25 students have a pet, what percent of the class has a pet?

Answers

Answer:

28%

Step-by-step explanation:

(25%-7%)×4

100%-28%

a local factory has to add a liquid ingredient to make there product at a rate of 13 quarts every 5 minutes. how many gallons per hour of the ingredient do they need to add

Answers

Per hour, they would need to add 147 quarts of the liquid ingredient. When converting to gallons, every 4 quarts is one gallon, so we need to divide 147 by 4.

36.75 Gallons Per Hour

They need to add 39 gallons of ingredients to make the production they desire.

What is a unitary method?

A unitary method is a mathematical way of obtaining the value of a single unit and then deriving any no. of given units by multiplying it with the single unit.

We know 4 quarts are one gallon.

∴ They would be making (13/4) = 3.35 gallons every 5 minutes Or (3.25/5) = 0.65 gallons per minute.

∴ In an hour or 60 minutes, they would be making (0.65×6) = 39 gallons per hour.

learn more about the unitary method here :

https://brainly.com/question/28276953

#SPJ2

Name two transformations that result in Figure ABCD being transformed back onto itself.

Answers

Answer:

  (b)  rotate 180°, then reflect across x and y axes

Step-by-step explanation:

A rotation of 180° cannot be "undone" by translation, or by reflection across the same line twice.

Reflection across the x- and y-axes, in either order, is equivalent to a rotation of 180° (or reflection across the origin). This pair of reflections will "undo" the rotation of 180°, putting the figure back in its original position

The transformation to map the figure to itself are ...

  rotation 180°, reflection across the x-axis, reflection across the y-axis


Please help me solve this problem

Answers

Answer:

15

Step-by-step explanation:

HELP PLEASE!! ;-; !! TWO PART QUESTION!! 1) Consider the image. Are the two triangles congruent if a is congruent to d and b is congruent to e?
A) Yes, by the ASA postulate.
B) Yes, by the SAS postulate.
Yes, by the SSS postulate.
D) No, they are not congruent.


2) suppose that three corresponding angles are congruent for two triangles (AAA), why can’t congruence be proved between the two triangles?
A) the triangles could have the same shape but not the same size.
B) The triangles could have the same size but not the same shape.
C) with three equal angles, only the shortest side of the triangles can be congruent.
D)With three equal angles, only the longest side of the triangles can be congruent.

Answers

Answer:

A

Step-by-step explanation:

A

i have two Questions.

1. Find the unit rate of 90 miles/2 hours, in feet per second.

2.Find the unit rate of 120 miles in 8 hours, in feet per second.


(extra credit: Extra Credit: How many unique handshakes can be made between 5 people?)

Answers

Step-by-step explanation:

ww have that 90 miles /2 hours is given and we are asked to find it in feet per second.

[tex] \frac{90 \: mi}{2 \: hrs} \times \frac{5280 \: ft}{1 \: mi} \times \frac{1 \: hr}{60 \: min} \times \frac{1 \: min}{60 \: sec} = 66[/tex]

So we get

66 fps.

2.

[tex] \frac{120 \: miles}{8 \: hours} \times \frac{5280}{1 \: mile} \times \frac{1 \: hour}{60 \: min} \times \frac{1 \: min}{60 \: seconds} = 22[/tex]

So we get 22 fps.

Extra credit There is 5 people and 2 hands so we use the combinations formula.

n!/r!(n-r)!. where n is the number of objects (the number of people) and r is the things taken at the time( the hands)

Subsitue

[tex] \frac{5 \times 4 \times 3 \times 2 \times 1}{2 \times 1 \times 3 \times 2 \times 1} [/tex]

[tex] \frac{20}{2} = 10[/tex]

Find 6889 + 3088 + 9062.

Answers

1903

pls mark brainliest

if u want

Answer:

19039

Step-by-step explanation:

hope that help mark me brinilylist

you need to explain more

it was easy

Lisa is getting paid 96$ per hour, her salary per hour increases by 12%, how much is she getting paid now after in increases by 12% ?

Answers

Answer:

96$/12×100

=8 × 100

=800$

Answer:

After the raise, I believe she is now getting paid 107.52$ per hour,

Step-by-step explanation:

12% of 96$ is 11.52$

11.52$ + 96$ = 107.52$

I am not sure if all calculations are correct, but I hope this helps! :)

Ahem please some help please
[tex]4 \div 0.5[/tex]

Answers

So 4 divided by 0.5 equals 8
8 is the Anserw cuz you need to put 0.5 divided by 4

Building For safety, the angle a wheelchair ramp makes with the horizontal should
be no more than 3.5º. What is the maximum height of a ramp of length 30 ft? What
distance along the ground would this ramp cover? Round to the nearest tenth of
a foot.

Answers

Answer:  8.6

Step-by-step explanation:

8.57142857143 -->     8.6

Solve geometry perimeter

Answers

7.62

Step-by-step explanation:

What two numbers multiplied together makes -15 then you add them together to make 2

Answers

Answer:

-3 and 5

Step-by-step explanation:

-3*5=-15

-3+5=2

Answer:

-3 and 5

Step-by-step explanation:

-3×5 I'd -15

also -3+5 is 2

Find the greatest common factor of 11a4
and
5y.

Answers

your answer is 1 :) hope this helps

Find the value of the expression:
x(x-y)-y(y^2-x) for x=4 and y=2

Answers

Replacing x by 4 and y by 2, it is found that the value of the expression is of 8.

The expression given is:

[tex]x(x - y) - y(y^2 - x)[/tex]

We want to evaluate it for x=4 and y=2, hence, we replace x by 4 and y by 2, then:

[tex]4(4 - 2) - 2(2^2 - 4) = 4(2) - 2(0) = 8[/tex]

The value of the expression is of 8.

A similar problem, in which the objective is finding the value of an expression, is given at https://brainly.com/question/19548262

Dalia's family bought 5 adult tickets and 2 student tickets to the school play for $65. Shay's family bought
3 adult tickets and 6 student tickets for $69. How much does each type of ticket cost?

Answers

Answer:

Lets say a is adult tickets and s is student tickets. for dalias family its:

5a + 2s = 65

for shay

3a + 6s = 69

-15a -6s = -195

-15a = -195

Step-by-step explanation:

calculate the height of the tree the answer is not 46​

Answers

We will want to use the tangent ratio for this question. Opposite=x, adjacent=30.5. Tan(33)=x/30.5.
Tan(33)(30.5)=x
X=19.806, or rounded off x=19.81.

Joe had some money. He spent $200 on clothes, 2/5 of his remaining money on a pair of boots and saved the rest. He saved $126. How much money did Joe have at first?​

Answers

Answer:

im sorry i dont get it

Step-by-step explanation:

Two plus two equal four

Answers

Answer:

Two plus two equals sixxxxxxx

The function P(t)=25,000(1.1023)t2 can be used to model the population in the town of Madison t years from now. Interpret the function in terms of the context for this situation.

Answers

Answer:

A growth function has the following form.

where  = Original amount,

r = annual rate of change,

n = number of periods,

t = time in years

Since the time is in years, we need to convert t into days. We know that 1 year = 365 days. So t years = 365t days.  

So in this case, n = 365, r = 0.0095.

Then the function is rewritten as follows.

The daily growth rate would be  

Step-by-step explanation:

t=number of years

Lets find year wise population for example

let years be 3t=3

[tex]\\ \sf{:}\longrightarrow P(3)=25,000(1.1023)3^2[/tex]

[tex]\\ \sf{:}\longrightarrow P(3)=27557.5(9)[/tex]

[tex]\\ \sf{:}\longrightarrow P(3)=248017.5[/tex]

Other Questions
PLEASE SOLVE THIS!!!6-3y+(-8y)+5.3 A machine takes 15 minutes to scan 1,125 pages. At this rate, how many pages can the machine in a hour? write the four super-family language that native african people speak. 3927283+8093Help pls James joins Club One which charges a monthly membership of $19.99. In 6 months, will he spend less than $120 in all? Please Help I will vote Brainliest! Hank has read 25 percent of his book. If Hank has read 150 pages, which equation can be used to find the total number of pages in the book?StartFraction 25 times 6 Over 100 times 6 EndFraction = StartFraction 150 Over 600 EndFractionStartFraction 100 times 4 Over 25 times 4 EndFraction = StartFraction 400 Over 100 EndFractionStartFraction 25 divided by 1 Over 150 divided by 1 EndFraction = StartFraction 25 Over 150 EndFractionStartFraction 150 divided by 2 Over 100 divided by 2 EndFraction = StartFraction 75 Over 50 EndFraction Solve.1.3=x(13.2)What is the solution to the equation?Enter your answer in the box.x = The y axis is _____________ horizontal vertical diagona Why did the United States support the Indian removal? An organism in its niche within an ecosystem is similar to?A: a baseball player in his position on the baseball team.B: the umpire not on a team.C: the audience in the stands of a game.D: the opposing team at a baseball game. Which composition of similarity transformations maps polygon abcd to polygon a'b'c'd'? a dilation with a scale factor of and then a rotation a dilation with a scale factor of and then a translation a dilation with a scale factor of 4 and then a rotation a dilation with a scale factor of 4 and then a translation. A plant uses the energy from sunlight to create oxygen and glucose from carbon dioxide and water which statement is true about the changes that have occurred A pet store holds training classes for dogs. The store has two different training programs. The first program charges a $35 membership fee and $5 for each class. The second program does not have a membership fee, but charges $10 for each class. For how many classes will the programs cost the same? If the variable c represents the number of classes, what is the other variable for the scenario? d, representing the number of dogs in each class h, representing the number of hours p, representing the number of programs t, representing the total cost. What is the 7th term in the sequence below?-19, -7, 5, 17, .....Select one:a.65b.29c.101d.53 Which is an example of gathering primary data?preparing a databasereviewing a historical reportperforming an Internet searchconducting a survey After Reconstruction ended in 1876, how did white Democrats achieve "Southern Redemption? 1. They provided funding to white supremacy groups such as the Ku Klux Klan.2. The repealed the Thirteenth Amendment and made slavery legal throughout the South. 3. They expanded many of the rights African Americans had earned during Reconstruction. 4. They restricted many of the rights African Americans had achieved during Reconstruction. Match each amendment with its description. 24 amendment opposite = 1.5. adjacent = 4. find the acute angle What would you have done if you had been Hari-Sarman? What is the image point of (1,4)(1,4) after a translation left 5 units and down 5 units?